18
9646/01/AJC2011Prelim [Turn Over 2011 JC2 Preliminary Examination PHYSICS 9646/01 Higher 2 Paper 1 Multiple Choice Wednesday 21 September 2011 1 hour 15 minutes Additional materials: Multiple Choice Answer Sheet READ THESE INSTRUCTIONS FIRST Write in soft pencil. Write your name and PDG in the spaces at the top of this page. Write your name, PDG, NRIC and shade the 7 digits of your NRIC/FIN in soft pencil on the Multiple Choice Answer Sheet. There are forty questions on this paper. Answer all questions. For each question there are four possible answers A, B, C and D. Choose the one you consider correct and record your choice in soft pencil on the separate Multiple Choice Answer Sheet. Each correct answer will score one mark. A mark will not be deducted for a wrong answer. Any rough working should be done in this question paper. This document consists of 18 printed pages. ANDERSON JUNIOR COLLEGE Candidate Name PDG ( )

2011 AJC Prelim H2 Physics P1

Embed Size (px)

DESCRIPTION

p1

Citation preview

Page 1: 2011 AJC Prelim H2 Physics P1

1

9646/01/AJC2011Prelim [Turn Over

2011 JC2 Preliminary Examination PHYSICS 9646/01 Higher 2 Paper 1 Multiple Choice Wednesday 21 September 2011 1 hour 15 minutes Additional materials: Multiple Choice Answer Sheet READ THESE INSTRUCTIONS FIRST

Write in soft pencil. Write your name and PDG in the spaces at the top of this page. Write your name, PDG, NRIC and shade the 7 digits of your NRIC/FIN in soft pencil on the Multiple Choice Answer Sheet. There are forty questions on this paper. Answer all questions. For each question there are four possible answers A, B, C and D. Choose the one you consider correct and record your choice in soft pencil on the separate Multiple Choice Answer Sheet. Each correct answer will score one mark. A mark will not be deducted for a wrong answer. Any rough working should be done in this question paper.

This document consists of 18 printed pages.

ANDERSON JUNIOR COLLEGE

Candidate Name

PDG ( )

Page 2: 2011 AJC Prelim H2 Physics P1

2

9646/01/AJC2011Prelim

Data speed of light in free space, c = 3.00 x 108 m s-1 permeability of free space, μ0 = 4π x 10-7 H m-1 permittivity of free space, ε0 = 8.85 x 10-12 F m-1 (1/(36π)) x 10-9 F m-1 elementary charge, e = 1.60 x 10-19 C the Planck constant, h = 6.63 x 10-34 J s unified atomic mass constant, u = 1.66 x 10-27 kg

rest mass of electron, me = 9.11 x 10-31 kg rest mass of proton, mp = 1.67 x 10-27 kg molar gas constant, R = 8.31 J K-1 mol-1 the Avogadro constant, NA = 6.02 x 1023 mol-1 the Boltzmann constant, k = 1.38 x 10-23 J K-1 gravitational constant, G = 6.67 x 10-11 N m2 kg-2

acceleration of free fall, g = 9.81 m s-2

Page 3: 2011 AJC Prelim H2 Physics P1

3

9646/01/AJC2011Prelim [Turn Over

Formulae

uniformly accelerated motion, 221 atuts +=

v2 = u2 + 2as work done on/by a gas, W = pΔV hydrostatic pressure, p = ρgh

gravitational potential, r

Gm = -φ

displacement of particle in s.h.m., x = x0 sin ωt velocity of particle in s.h.m., v = v0 cos ωt

220 xxωv −±=

mean kinetic energy of a kT23E =

molecule of an ideal gas, resistors in series, R = R1 + R2 + … resistors in parallel, 1/R = 1/R1 + 1/R2 + …

electric potential, r4

Q = V0πε

alternating current/voltage, x = x0 sin ωt

transmission coefficient, T ∝ exp(-2kd)

where k = ( )2

28h

EUm −π

radioactive decay, x = x0exp(- λt)

decay constant, 21t

0.693 = λ

Page 4: 2011 AJC Prelim H2 Physics P1

4

9646/01/AJC2011Prelim

1

A quantity X is to be determined from the equation

X = P – Q given that P = 1.27 ± 0.02 m. Q = 0.83 ± 0.01 m. What is the percentage uncertainty in X?

A 0.4 % B 2 % C 3 % D 7 % 2

Errors in measurement may be either systematic or random. Which of the following involves random errors? A not allowing for zero error on a voltmeter B not subtracting background count rate when determining the count rate from a radioactive

source C stopping a stopwatch at the end of a race D using the value of g as 10 N kg-1 when calculating weight from mass

3

In order that a train can stop safely, it passes a signal showing a yellow light before it reaches another signal showing a red light. Drivers apply the brake at the yellow light and this results in a uniform deceleration to stop exactly at the red light. The distance between the red and yellow lights is x. What must be the minimum distance between the lights if the train speed is increased by 20%, without changing the deceleration of the trains?

A 1.20x B 1.25x C 1.44x D 1.56x

Page 5: 2011 AJC Prelim H2 Physics P1

5

9646/01/AJC2011Prelim [Turn Over

4

A student throws a ball vertically upwards. Upward velocities are taken as positive. The ball makes an elastic collision with the ceiling, rebounds and accelerates back to the student’s hand in a time of 1.2 s. Which graph best represents the acceleration of the ball from the moment it leaves the hand to the instant just before it returns to the hand?

A

B

C

D

5

The diagram shows two spherical masses approaching each other head-on at an equal speed u. One has mass 2m and the other has mass m.

Which diagram represents the situation after an elastic collision?

A

B

C

D

Page 6: 2011 AJC Prelim H2 Physics P1

6

9646/01/AJC2011Prelim

6

A box of mass 8.0 kg rests on a horizontal, rough surface. A string attached to the box passes over a smooth pulley and supports a 2.0 kg mass at its other end.

When the box is released, a friction force of 6.0 N acts on it. What is the acceleration of the box?

A 1.4 m s-2 B 1.7 m s-2 C 2.0 m s-2 D 2.5 m s-2 7

The diagram shows a metal cube suspended from a spring balance before and during immersion in water.

A reduction in the balance reading occurs as a consequence of the immersion. Which statement is correct? A The balance reading will be further reduced if the cube is lowered further into the water. B The balance reading during immersion corresponds to the upthrust of the water on the

cube. C The forces acting on the vertical sides of the cube contribute to the change in the balance

reading. D The gravitational pull on the cube is unchanged by the immersion.

Page 7: 2011 AJC Prelim H2 Physics P1

7

9646/01/AJC2011Prelim [Turn Over

8

A rigid L-shaped lever arm is pivoted at point P.

Three forces act on the lever arm, as shown in the diagram. What is the magnitude of the resultant moment of these forces about point P?

A 15 Nm B 20 Nm C 35 Nm D 75 Nm 9

The diagram shows two identical vessels X and Y connected by a short pipe with a tap.

Initially, X is filled with water of mass m to a depth h, and Y is empty. When the tap is opened, water flows from X to Y until the depths of water in both vessels are equal. How much potential energy is lost by the water during this process? (g = acceleration of free fall)

A 0 B

4mgh

C 2

mgh D mgh

Page 8: 2011 AJC Prelim H2 Physics P1

8

9646/01/AJC2011Prelim

10

Two trolleys are placed together on a horizontal runway with a compressed spring between them.

When they are released, the 2 kg trolley moves to the left at 2 m s-1. How much energy was stored in the spring?

A 4 J B 6 J C 8 J D 12 J 11 A mass on the end of a string is set in motion so that it describes a circle in a horizontal

plane at a constant speed. The angle made by the string to the vertical is 40°. The only forces acting on the mass are the tension T and its weight W. What is the ratio of the resultant force to the weight of the mass?

A 0.012 B 0.643 C 0.766 D 0.839 12 What is the speed of a geostationary satellite orbiting Earth?

(mass of the Earth = 6.0 × 1024 kg)

A 3.08 × 103 m s−1 B 4.71 × 104 m s−1 C 1.80 × 105 m s−1 D 4.23 × 107 m s−1 13

What is meant by the weight of an object? A the gravitational field acting on the object B the gravitational force acting on the object C the mass of the object multiplied by gravity D the object’s mass multiplied by its acceleration

W

T

40°

Page 9: 2011 AJC Prelim H2 Physics P1

9

9646/01/AJC2011Prelim [Turn Over

14

The diagram shows points W, X, Y and Z on a line joining the centre of the Earth to the centre of the Moon. W and Z are on the surfaces of the Earth and Moon respectively. X is the mid-point between the Earth and the Moon. Y is a point where an object will not experience any resultant force. A spacecraft moves between the Earth and the Moon along this line. Which transition would give the greatest change in gravitational potential energy of the spacecraft?

A W to X B W to Y C W to Z D Y to Z 15

The diagram shows a velocity-time graph for a mass moving up and down on the end of a spring. Taking velocity upward to be positive, which point represents the velocity of the mass when at the lowest point of its motion?

W X Y

Earth Moon

Z

Page 10: 2011 AJC Prelim H2 Physics P1

10

9646/01/AJC2011Prelim

16 The diagram below shows the variation with frequency f of the amplitude x0 of the forcedoscillation of a machine. At resonance, calculate the maximum linear velocity and state whether the machine experience any damping.

velocity / m s-1 damping

A

B

C

D

1.55

1.55

14.6

14.6

No

Yes

No

Yes 17

Which phenomenon is associated with transverse waves but not longitudinal waves?

A rarefaction B interference C polarisation D reflection

0 0.5 1.0 1.5 2.0 2.5 3.0 f / Hz

x0 / cm 16 14

12 10

8 6

4 2 0

Page 11: 2011 AJC Prelim H2 Physics P1

11

9646/01/AJC2011Prelim [Turn Over

18

A health inspector is measuring the intensity of a sound. Near a loudspeaker his meter records an intensity I. This corresponds to an amplitude A of the sound wave. At another position the meter gives an intensity reading of 2I. What is the corresponding sound wave amplitude?

A

2A B 2A C 2A D 4A

19 A wire of length L fixed at both ends is placed vertically between two magnets of opposite

poles. A stationary wave is set up in the wire as shown in the diagram when an alternating voltage of frequency 50 Hz is applied across the wire. Which of the following is false? A The wire will not vibrate if the voltage is a direct source. B The greatest amplitude is obtained when the wavelength of the stationary wave set up in wire is 2L. C The wire would vibrate at 50 Hz, regardless of the strength of the magnetic field applied by the magnet. D The wire will set up a stationary wave at when the frequency of the alternating voltage is reduced to 25 Hz.

20 A parallel beam of white light is incident normally on a diffraction grating. It is noted that the

second-order and third-order spectra partially overlap. Which wavelength in the third-order spectrum appears at the same angle as the wavelength of 600 nm in the second-order spectrum?

A 300 nm

B 400 nm

C 600 nm

D 900 nm

21 When one junction X of a thermocouple is placed in melting ice and the other junction Y in

steam at 100 °C, the e.m.f. is 8.0 mV. Junction X is removed from the melting ice and is placed in a liquid bath at a constant temperature and junction Y is removed from the steam and placed in water at a constant temperature of 40 °C. The e.m.f. is now -1.6 mV. What is the temperature of the bath on the centigrade scale of this thermocouple?

A 20 °C B 60 °C C 80 °C D 120 °C

S

N

Page 12: 2011 AJC Prelim H2 Physics P1

12

9646/01/AJC2011Prelim

22 A sample of an ideal gas may (i) compress at constant temperature, or (ii) compress at constant pressure. The net flow of heat into the gas from the exterior is A negative in each case B positive in each case C negative for (i) and positive for (ii) D positive for (i) and negative for (ii)

23 Two identical vessels contain two different gases X and Y with molecular mass mx and my

respectively. The absolute temperature of gas X is twice that of gas Y and the root-mean-square speed of the molecules in X is three times that of the molecules in Y. What is the

ratio of X

Y

mm

.

A 2 : 3 B 2 : 9 C 9 : 2 D 3 : 2 24 The diagram shows a simple potentiometer circuit for measuring the e.m.f. of cell Q using a

driver cell P and wire XY. However, no balance point could be found in this set-up. A balance point might be obtained by A replacing cell P by a cell of greater e.m.f. B replacing wire XY by a wire with a greater cross-sectional area. C adding a resistor in series with cell P. D adding a resistor in series with cell Q.

P

Q

X Y

galvanometer

Page 13: 2011 AJC Prelim H2 Physics P1

13

9646/01/AJC2011Prelim [Turn Over

25 Two cells of e.m.f E1 and E2 with negligible internal resistance are connected with two variable resistors as shown in the diagram.

When the galvanometer deflection is zero, the resistances of the variable resistors are P

and Q. What is the value of the ratio 1

2

EE

?

A

QP B

QPP+

C QP

Q+

D P

QP +

26 The circuit shown in the diagram below is connected to a power supply and the ammeter A1

reads 6.0 A. Assuming that both ammeters have negligible resistance, what is the reading on ammeter A2?

A 2.3 A B 4.5 A C 6.0 A D 13.5 A 27 Two parallel conducting plates are connected to a battery, one plate to the positive terminal

and the other plate to the negative. The plate separation is gradually increased, the plates remaining connected to the battery. Which graph shows how the electric field E between the plates depends on the plate separation x? A B C D

A1 A2 4R

2R

3R

R

Page 14: 2011 AJC Prelim H2 Physics P1

14

9646/01/AJC2011Prelim

28 The diagram shows equipotential lines spaced 2.0 cm apart. The electric force exerted on a charge of −5.0 µC when placed at X is A 3.0 x 10-2 N to the right B 3.0 x 10-2 N to the left C 1.5 x 10-2 N to the right D 1.5 x 10-2 N to the left

29 A bent wire PQRST carrying a current I is placed in a magnetic field of flux density B as

shown. QR is the shortest and ST is the longest. PQ and RS are equal in length and both are longer than QR. The forces acting on PQ, QR, RS and ST are given by FPQ, FQR, FRS and FST respectively. Which of the following is true? A FQR ≠ FRS and FPQ < FST B FQR = FRS and FPQ = FST C FQR > FRS and FPQ < FST D FQR < FRS and FPQ = FST

equipotential line

80 V -100V20 V

2.0 cm 2.0 cm 2.0 cm

X

-40 V

B P Q

R

S T I

Page 15: 2011 AJC Prelim H2 Physics P1

15

9646/01/AJC2011Prelim [Turn Over

30 A particle of mass m and charge +q enters with speed v at right angles to a magnetic field of flux density B. The particle arrives at point P, a distance d from the slit as shown in the diagram below. Which of the following is a correct expression for d?

A Bq

m24π B

Bqmv2

C Bqmv2

D Bqmv8

31 A rectangular coil is rotating with constant angular velocity in a magnetic field.

Which one of the following statements is false? A The maximum induced e.m.f. will increase if the frequency of rotation is increased. B The maximum induced e.m.f. will increase if the cross sectional area for the coil is increased. C The maximum induced e.m.f. will increase if the Earth’s magnetic field is in the same direction as the applied magnetic field. D The maximum induced e.m.f. will increase if the axis of rotation is parallel to the magnetic field.

axis of rotation

B d

v P

Page 16: 2011 AJC Prelim H2 Physics P1

16

9646/01/AJC2011Prelim

32 A rectangular loop of wire enters a region of uniform magnetic field and leaves the field at a uniform speed v. The magnetic field is perpendicular to the plane of the loop of wire. Which graph best shows the variation with time t of the current I induced in the frame? A B

C D

t

Page 17: 2011 AJC Prelim H2 Physics P1

17

9646/01/AJC2011Prelim [Turn Over

33 The diagram shows the variation with time of a periodic current. The current of peak value I0 passes through a heater of resistance R. What is the mean power output of the heater?

A 4

20 RI

B 2

0I2R C

20I

2R D R2

0I

34 A metal surface in an evacuated tube is illuminated with monochromatic light causing the

emission of photoelectrons which are collected at an adjacent electrode. If the experiment is to be repeated with light of half the intensity but the same wavelength, how will the photocurrent I and stopping potential V be affected? A I unchanged, V unchanged B I unchanged, V halved C I halved, V unchanged D I halved, V halved

35 The following graph shows the spectrum of X-rays emitted from an X-ray tube.

If the potential difference between the target and cathode is increased, which one of the following combinations represents a possible change in wavelength and intensity of the peaks?

wavelength intensity A

B

C

D

remain the same

decrease

remain the same

decrease

increase

remain the same

remain the same

increase

intensity

wavelength

Io I/A

t/ms

Page 18: 2011 AJC Prelim H2 Physics P1

18

9646/01/AJC2011Prelim

36 A proton has a kinetic energy of 1.00 MeV. If its momentum is measured with an uncertainty of 1.00 %, what is the minimum uncertainty in its position?

A 1.95 × 10−11 m B 4.57 × 10−13 m C 9.08 × 10−14 m D 6.59 × 10−20 m 37 The resistance of a piece of pure silicon falls as the temperature rises. Which statement is

true? A The ratio of the positive to negative charge carriers increases. B The ratio of the positive to negative charge carriers decreases. C The charge carriers can move more easily at a higher temperature. D The total number of charge carriers increases with temperature.

38 The word LASER comes from the first letters of the words in the phrase Light Amplification

by the Stimulated Emission of Radiation. What is meant by stimulated emission? A an electron from a higher level falling to a lower level B a charged particle causing light to be emitted from an excited atom C a charged particle being emitted from an atom as a result of a high energy photon hitting

the atom. D a photon causing another photon of the same frequency to be emitted from an excited

atom

39 Radon 222

86Rn decays by α− and β−emission to bismuth 21483Bi .

For the decay of each nucleus of radon, how many α− and β−particles are emitted?

α− particles β−particles

A

B

C

D

1

2

1

2

1

1

2

2

40 The activity from a radioactive source is found to fall by 0.875 of its initial activity in 210 s.

What is the half-life of the source?

A 30 s B 70 s C 210 s D 1100 s